Linear Transformations: Finding Matrix with Standard Basis

Click For Summary
The discussion centers on finding the matrix representation of the linear transformation T: P3 → P3 defined by T(p(x)) = p(x+1) + p(2-x) with respect to the standard basis {1, x, x^2, x^3}. The user successfully computes T for each basis element, yielding T(1) = 2, T(x) = 3, T(x^2) = 2x^2 - 2x + 5, and T(x^3) = 9x^2 - 9x + 9. Guidance is provided to express these transformations as column vectors, which will form the columns of the transformation matrix. The user confirms understanding and expresses gratitude for the assistance. The conversation effectively clarifies the process of constructing the matrix from the linear transformation results.
baddin
Messages
24
Reaction score
0
1. Give information
Let T: P3 ---> P3 be the linear transformation described by:
T(p(x))=p(x+1)+p(2-x).
Find the matrix of T with respect to the standard basis b {1,x,x^2,x^3}.


The Attempt at a Solution


I found the transformations on the standard basis b:
T(1) = 2
T(x) = 3
T(x^2) = 2x^2 -2x +5
T(x^3) = 9x^2 - 9x + 9
I am confused on what to do next...
 
Physics news on Phys.org
baddin said:
1. Give information
Let T: P3 ---> P3 be the linear transformation described by:
T(p(x))=p(x+1)+p(2-x).
Find the matrix of T with respect to the standard basis b {1,x,x^2,x^3}.


The Attempt at a Solution


I found the transformations on the standard basis b:
T(1) = 2
T(x) = 3
T(x^2) = 2x^2 -2x +5
T(x^3) = 9x^2 - 9x + 9
I am confused on what to do next...

Write your functions so they look a little more like vectors, write a+bx+cx^2+dx^3 as the column vector [a,b,c,d]. So T(1)=2 becomes T([1,0,0,0])=[2,0,0,0]. Does that help?
 
Ok, then I should find T(1,0,0,0), T(0,1,0,0), T(0,0,1,0) and T(0,0,0,1) right?
 
baddin said:
Ok, then I should find T(1,0,0,0), T(0,1,0,0), T(0,0,1,0) and T(0,0,0,1) right?

Right. You really already did. Just write them as column vectors. Then those will be the columns of your matrix.
 
  • Like
Likes 1 person
Okay thank you very much for your help =)
 
Question: A clock's minute hand has length 4 and its hour hand has length 3. What is the distance between the tips at the moment when it is increasing most rapidly?(Putnam Exam Question) Answer: Making assumption that both the hands moves at constant angular velocities, the answer is ## \sqrt{7} .## But don't you think this assumption is somewhat doubtful and wrong?

Similar threads

  • · Replies 2 ·
Replies
2
Views
1K
  • · Replies 3 ·
Replies
3
Views
2K
  • · Replies 5 ·
Replies
5
Views
2K
  • · Replies 18 ·
Replies
18
Views
2K
  • · Replies 10 ·
Replies
10
Views
3K
  • · Replies 6 ·
Replies
6
Views
2K
  • · Replies 11 ·
Replies
11
Views
2K
  • · Replies 4 ·
Replies
4
Views
1K
  • · Replies 4 ·
Replies
4
Views
2K
  • · Replies 10 ·
Replies
10
Views
2K